初 等 整 数 論 講 義 第 $2$ 版

$\blacktriangleleft$ $\S\ 58.$ イデヤルの類の数 $h$ の計算  $\S\ 60.$ Gauss の和 $\blacktriangleright$

『初等整数論講義 第 $2$ 版』目次へ



附      録

 $\S\ 59.$ 算術級数中の素数

 $\boldsymbol{1.}$ $\boldsymbol{k}$ は正の有理整数で $\boldsymbol{\left(k,\ m\right)=1}$ ならば\[\boldsymbol{p\equiv m\hphantom{n}\left(\text{mod}.\ k\right)}\]なる素数 $\boldsymbol{p}$ が無数に存在する$21$ 頁).
 この Dirichlet の定理はたびたび引合いには出したが,まだその証明を述べなかったので,この定理を利用することができなかったのである.しかし初等整数論でも,この定理を応用することが便利である機会ははなはだ多いから,いまここで Dirichlet の証明法の概略を述べておく.その証明が初等的でないのが遺憾であるが,数学の現状ではやむを得ないのである.
 $\boldsymbol{2.}$ $\text{mod}.\ k$ に関する指標を $\chi\left(n\right)$ として\[L\left(s,\ \chi\right)=\overset{\infty}{\underset{n=1}{\textstyle\sum}}\frac{\chi\left(n\right)}{n^s}\]なる級数で表わされる $s$ の函数を Dirichlet の $L$ 函数という.
 Dirichlet の証明法は $s=1$ における $L$ 函数の性状に基づく.すなわち次の通り.
 $\left[\ 1\ \right]$  $L$ 函数は $s\gt1$ であるとき絶対収束をして,次のように無限積として表わされる:\[L\left(s,\ \chi\right)=\overset{p}{\textstyle\prod}\frac{1}{1-\dfrac{\chi\left(p\right)\ \!}{p^s}}.\tag{$\ 1\ $}\]積はすべての有理素数 $p$ の上にわたる.
 これは $\S\ 58$ の $\zeta\left(s\right)$,$L_K\left(s\right)$ と同様であるから,証明を省略する.
 $\text{mod}.\ k$ に関する $\varphi\left(k\right)$ 個の指標 $\chi\left(n\right)$ に応じて,同数の $L$ 函数が生ずるが,指標の三種類に対応して $L$ 函数を三種に分け,一般的に,それを $L_1$,$L_2$,$L_3$ で表わす.
 $L_1$ は第一種の指標に対応するもので,それはただ一つである.第二種の指標に対応する $L_2$ は $k$ に含まれる相異なる素数の数を $t$ とすれば,その数 $2^t-1$ または $2^{t+1}-1$ である(指数 $t+1$ は,$k$ が $8$ で割り切れる場合).
 第三種の指標に対応する $L_3$ は指標 $\chi\left(n\right)$ と逆指標 $\overline{\chi}\left(n\right)=\chi\left(n\right)^{-1}$ とに対応して二つずつ互いに共軛である.それらを $L_3$,$\overline{L}_3$ で表わすことにする.
 $\left[\ 2\ \right]$  $\underset{s\rightarrow1}{\operatorname{Lim}}\ \!\!\!\left(s-1\right)L_1\left(s\right)=\dfrac{\varphi\left(k\right)}{k}$.
 第一種指標 $\chi_1\left(n\right)$ は $\left(n,\ k\right)\neq1$ なる $n$ に対して $0$ であるほかはすべて $1$ に等しい.故に\[L_1\left(s\right)=\overset{p}{\textstyle\prod}{}^\prime\frac{1}{1-\dfrac{1}{p^s}}\]で,積 $\prod{}^\prime$ は $k$ に含まれないすべての素数 $p$ の上にわたる.故に\[L_1\left(s\right)=\overset{p\ |\ k}{\textstyle\prod}\left(1-\frac{1}{p^s}\right)\zeta\left(s\right),\]$\zeta\left(s\right)$ は Riemann の函数($375$ 頁)で,$\displaystyle\overset{p|k}{\textstyle\prod}$ は $k$ に含まれる素数 $p$ に関する有限積である.故に\begin{alignat*}{1}\underset{s\rightarrow1}{\operatorname{Lim}}\left(s-1\right)L_1\left(s\right)=&\overset{p\ |\ k}{\textstyle\prod}\left(1-\frac{1}{p}\right)\underset{s\rightarrow1}{\operatorname{Lim}}\left(s-1\right)\zeta\left(s\right)\\[2mm]&=\frac{\ \!\varphi\left(k\right)\ \!\!}{k}.\end{alignat*} $\left[\ 3\ \right]$  $L_2\left(s\right)$,$L_3\left(s\right)$ は($s\gt1$ のとき絶対的に収束し)$s=1$ の附近で一様収束をするから,$s=1$ において連続的である.したがって\[\operatorname{Lim}\ \!\!L_2\left(s\right)=L_2\left(\ 1\ \right),\hphantom{n}\operatorname{Lim}\ \!\!L_3\left(s\right)=L_3\left(\ 1\ \right)\]であるが,ここに大切なのは\[L_2\left(\ 1\ \right)\neq0,\hphantom{n}L_3\left(\ 1\ \right)\neq0.\] $L_2\left(s\right)$ に関しては収束問題は $\S\ 58$($378$ 頁)と全く同様である.さて $L_2\left(\ 1\ \right)\neq0$ は次に述べるように $\S\ 58$ からすぐにわかる.それが Dirichlet の方法の妙所である.
 いま $k$ を素数巾に分解して\[k=l_1{}^{e_1}\ldots\ldots l_\nu{}^{e_\nu}=\textstyle\prod l^e\]とする.しからば $k$ が奇数ならば($120$ 頁),\[n\equiv a_1{}^{i_1}a_2{}^{i_2}\ldots\ldots a_\nu{}^{i_\nu}\hspace{1cm}\left(\text{mod}.\ k\right)\]とするとき,第二種指標は\[\chi_2\left(n\right)=\left(\pm1\right)^{i_1}\left(\pm1\right)^{i_2}\ldots\ldots\left(\pm1\right)^{i_\nu}.\] ここでおのおのの $l$ に対応する $\pm1$ の中 $-1$ になるものだけをとって,それらは $l_1$,$l_2$,$\ldots\ldots$,$l_\mu$($\mu\leqq\nu$)に対応するとする.しからば\[d=\pm l_1l_2\cdots\cdots l_\mu\equiv1\hphantom{4}\left(\text{mod}.\ 4\right)\]とおけば,$\chi_2\left(n\right)$ は Kronecker の記号 $\chi\left(n\right)=\left(\dfrac{d}{n}\right)$ と同様であるが,ただ $l_{\mu+1}$,$l_{\mu+2}$,$\cdots\cdots$,$l_\nu$ で割り切れる $n$ に対しては $\chi_2\left(n\right)=0$ になるだけが違う.故に $\left[\ 2\ \right]$ と同様に\[L_2\left(s\right)={\textstyle\prod}\left(1-\frac{\chi\left(l\right)}{l^s}\right)L_K\left(s\right),\]$\prod$ は $l=l_{\mu+1}$,$l_{\mu+2}$,$\ldots\ldots$,$l_\nu$ に関する有限積で,$L_K\left(s\right)$ は $\S\ 58$ の通り.故に\[L_2\left(\ 1\ \right)={\textstyle\prod}\left(1-\frac{\chi\left(l\right)}{l}\right)L_K\left(\ 1\ \right)\]であるが,$\S\ 58$ によって $L_K\left(\ 1\ \right)=h\kappa\neq0$.故に $L_2\left(\ 1\ \right)\neq0$.
 上記では $k$ を奇数としたが,$k$ が偶数で $4$ または $8$ で割り切れるときには,上記 $d$ へ $4$ または $8$ の因数を付け加えるべき場合もある.
 さて $L_3\left(s\right)$ に関しても収束問題は $\S\ 58$($377$ 頁,$\left[\ 3\ \right]$)と同様である.しかし,ここでは $L_3\left(s\right)$ を複素変数 $s=\sigma+ti$ の函数と見るのがよい,すなわち\[L_3\left(s\right)=\overset{\infty}{\underset{n=1}{\textstyle\sum}}\frac{\chi_3\left(n\right)}{n^s}=\overset{\infty}{\underset{n=1}{\textstyle\sum}}\chi_3\left(n\right)e^{-s\log n}\] 複素数平面上 $s=1$ の附近で,$L_3\left(s\right)$ が一様に収束するから,函数論でよく知られた定理(Weierstrass の定理)によって $L_3\left(s\right)$ は $s=1$ の附近で正則である.
 事実上は $L_3\left(s\right)$ および $L_2\left(s\right)$ は超越的整函数であるが,われわれの当面の目的には必要がないから省略する.
 故に $L_3\left(s\right)$ と $\overline{L}_3\left(s\right)$ とに関しては,$s=1$ において次のような展開が可能である:\[\begin{array}{l}\hphantom{\log_3}\begin{alignat*}{1}&L_3\left(s\right)=\left(s-1\right)^\nu\left\{c+c^\prime\left(s-1\right)+\cdots\cdots\right\},\\[2mm]&\overline{L}_3\left(s\right)=\left(s-1\right)^\nu\left\{\overline{c}+\overline{c}{}^\prime\left(s-1\right)+\cdots\cdots\right\},\end{alignat*}\hspace{15mm}c\neq0\\[2mm]\hphantom{\log_3}L_3\left(s\right)\overline{L}_3\left(s\right)=\left(s-1\right)^{2\nu}\{\left|c\right|{}^2+d\left(s-1\right)+\cdots\cdots\},\\[2mm]\log L_3\left(s\right)\overline{L}_3\left(s\right)=2\nu\log\left(s-1\right)+g\left(s\right)\end{array}\]で,$g\left(s\right)$ は $s=1$ において正則である.$L_3\left(\ 1\ \right)\neq0$ または $L_3\left(\ 1\ \right)=0$ に従って $\nu=0$,または $\nu\geqq1$ であるから,$\nu=0$ を示せば,$\left[\ 3\ \right]$ の後半が証明される.それは次のような間接法でできる.

 $\boldsymbol{3.}$ いますべての $\chi$ に関して $\log L\left(s,\ \chi\right)$ の和を作って $\left[\ 2\ \right]$,$\left[\ 3\ \right]$ によって\[\overset{\chi}{\textstyle\sum}\log L\left(s,\ \chi\right)=(\textstyle\sum2\nu-1)\log\left(s-1\right)+f\left(s\right)\tag{$\ 2\ $}\]とおけば,$s\rightarrow1$ のとき $f\left(s\right)$ は有限である.
 しかるに $\left(\ 1\ \right)$ から\[\log L\left(s,\ \chi\right)=\overset{p}{\textstyle\sum}\frac{\chi\left(p\right)}{p^s}+\frac{1}{2}\overset{p}{\textstyle\sum}\frac{\chi\left(p^2\right)}{p^{2s}}+\cdots\tag{$\ 3\ $}\]で,右辺における第二以下の $\sum$ の総和は $s\geqq1$ のとき絶対値において $1$ よりも小である($384$ 頁).また\[\overset{\chi}{\textstyle\sum}\chi\left(p\right)=\left\{\begin{array}{cl}\varphi\left(k\right),&\hspace{1cm}p\equiv1\hphantom{m}\left(\text{mod}.\ k\right)\\[2mm]0,&\hspace{1cm}p\not\equiv1\hphantom{m}\left(\text{mod}.\ k\right)\end{array}\right.\]だから\[\overset{\chi}{\textstyle\sum}\log L\left(s,\ \chi\right)=\varphi\left(k\right){\textstyle\sum}{}^\prime\frac{1}{p^s}+S\tag{$\ 4\ $}\]とおけば,$s\geqq1$ のとき $\left|S\right|\lt\varphi\left(k\right)$ で,${\textstyle\sum}{}^\prime$ は $p\equiv1\hphantom{k}\left(\text{mod}.\ k\right)$ なる素数 $p$ の上にわたる.
 さて,もしも或る $L_3\left(s\right)$ に関して $L_3\left(\ 1\ \right)=0$ と仮定するならば,$\nu\geqq1$ であるから,$\left(\ 2\ \right)$ によって $s\rightarrow1$ のとき,$\sum\log L\left(s,\ \chi\right)\rightarrow-\infty$.しかるに $\left(\ 4\ \right)$ によれば,これは不可能である.故に $L_3\left(\ 1\ \right)\neq0$ である.したがって $\left[\ 3\ \right]$ が証明されたのであるが,$\left(\ 2\ \right)$ の右辺で $\sum2\nu=0$ とすれば,$\left(\ 4\ \right)$ と比較して\[{\textstyle\sum}{}^\prime\frac{1}{p^s}\sim\frac{1}{\varphi\left(k\right)}\log\frac{1}{s-1}.\]すなわち $s\rightarrow1$ のとき,$\sum{}^\prime\dfrac{1}{p^s}\rightarrow\infty$,故に $p\equiv1\left(\text{mod}.\ k\right)$ なる素数 $p$ が無数に存在しなければならない.
 これは,しかしながら,すでに $\S\ 10$ で初等的の方法で証明したのであった.
 $\boldsymbol{4.}$ $p\equiv m\hphantom{k}\left(\text{mod}.\ k\right)$ なる素数 $p$ の存在することは次のようにして証明される.$\log L\left(s,\ \chi\right)$ の和を作る代わりに,それに $\overline{\chi}\left(m\right)$ を掛けて後に和を作れば\[\overset{\chi}{\textstyle\sum}\overline{\chi}\left(m\right)\log L\left(s,\ \chi\right)=\overset{\chi,\ p}{\textstyle\sum}\frac{\overline{\chi}\left(m\right)\chi\left(p\right)}{p^s}+S^\prime.\] ここでも $s\geqq1$ のとき $\left|S^\prime\right|\lt\varphi\left(k\right)$ である.さて\[{mm}^\prime\equiv1\hphantom{k}\left(\text{mod}.\ k\right)\]とすれば\[\overline{\chi}\left(m\right)=\chi\left(m^\prime\right),\hspace{5mm}\chi\left(m^\prime\right)\chi\left(p\right)=\chi\left({pm}^\prime\right).\] 故に\begin{alignat*}{1}\overset{\chi,\ p}{\textstyle\sum}\frac{\overline{\chi}\left(m\right)\chi\left(p\right)}{p^s}&=\overset{p}{\textstyle\sum}\frac{\overset{\chi}{\sum}\chi\left({pm}^\prime\right)}{p^s}\\[2mm]&=\varphi\left(k\right)\overset{p}{\textstyle\sum}{}^\prime\frac{1}{p^s}.\end{alignat*}ここでは $\sum{}^\prime$ は ${pm}^\prime\equiv1\hphantom{k}\left(\text{mod}.\ k\right)$,すなわち $p\equiv m\hphantom{k}\left(\text{mod}.\ k\right)$ なるすべての素数 $p$ の上にわたるのである.すなわち\[\overset{\chi}{\textstyle\sum}\overline{\chi}\left(m\right)\log L\left(s,\ \chi\right)=\varphi\left(k\right){\textstyle\sum}{}^\prime\frac{1}{p^s}+S^\prime.\]しかるに一方において,\[\overset{\chi}{\textstyle\sum}\overline{\chi}\left(m\right)\log L\left(s,\ \chi\right)=-\log\left(s-1\right)+f_2\left(s\right)\]と置けば,$\overline{\chi}_1\left(m\right)=\chi_1\left(m\right)=1$ に注意して $\left[2\right]$,$\left[3\right]$ から $s\rightarrow1$ のとき $f_2\left(s\right)$ は有限である.よって\[\overset{p}{\textstyle\sum}{}^\prime\frac{1}{p^s}\sim\frac{1}{\varphi\left(k\right)}\log\frac{1}{s-1}.\]故に $p\equiv m\hphantom{k}\left(\text{mod}.\ k\right)$ なる素数 $p$ は無数に存在しなければならない.
 上記 Dirichlet の証明は $L_2\left(1\right)\neq0$,$L_3\left(1\right)\neq0$ に基づくのであるが,$L_2\left(1\right)\neq0$ は二次体のイデヤルの類の数 $h$ の計算の副産物として出てくることが Dirichlet の方法を成功せしめた原因である.$L_3\left(1\right)\neq0$ も容易に証明はされたが,しかしもしも $1$ の $k$ 乗根から生ずる数体のイデヤル論によるならば,そのイデヤルの類の数の計算によって,$L_3\left(1\right)\neq0$ が自然に出てしまうのである.このように算術級数の定理と $1$ の巾根の整数論との間に密接な関係のあることは,興味ある著しい事実といわねばなるまい($\S\ 10$ における $m=1$ の場合の証明が,すでにその一端を示している).

 しかし一方において算術級数の定理だけを切り離して考察するならば,それをイデヤル論から独立して,直接に証明することも興味ある問題である.Dirichlet の後 Mertens が直接に $L_2\left(1\right)\neq0$ を証明して二次体論の統制を脱し,近時 Landau がさらにその証明を短縮したが,これらは要するに Dirichlet の証明法の変形にすぎない.

 算術級数の定理を真に直接に(イデヤル論からも,函数論からも,独立に)証明することは未解の謎である.






$\blacktriangleleft$ $\S\ 58.$ イデヤルの類の数 $h$ の計算  $\S\ 60.$ Gauss の和 $\blacktriangleright$

『初等整数論講義 第 $2$ 版』目次へ


 ページトップへ inserted by FC2 system